A flagpole 94.9 ft tall is on the top of a building. From a point on level​ ground, the angle of elevation of the top of the flagpole is 35.3degrees​, while the angle of elevation of the bottom of the flagpole is 26.2degrees. Find the height of the building.

Answers

Answer 1

Answer:

214 ft

Step-by-step explanation:

Height of building = 94.9 ft

The angle of elevation of the top of the flagpole = θ₁ = 35.3°

The angle of elevation of the bottom of the flagpole = θ₂ = 26.2°

Let,

Height of building = x

Distance from observation point to base of building = y

[tex]tan 26.2 =\frac{x}{y}\\\Rightarrow y=\frac{x}{tan26.2}[/tex]

[tex]tan 35.3 =\frac{94.9+x}{y}\\\Rightarrow tan 35.3 =\frac{94.9+x}{\frac{x}{tan26.2}}\\\Rightarrow \frac{x}{tan26.2}tan35.3=94.9+x\\\Rightarrow \frac{tan35.3}{tan26.2}x-x=94.9\\\Rightarrow x=\frac{94.9}{\frac{tan35.3}{tan26.2}-1}\\\Rightarrow x=214.84/ ft[/tex]

I have used the exact values from the calculator.

∴ Height of the building is 214.84 ft

A Flagpole 94.9 Ft Tall Is On The Top Of A Building. From A Point On Level Ground, The Angle Of Elevation
Answer 2

Answer:

The height of the building is 214.84 ft.

Step-by-step explanation:

Given information:

The height of the flagpole = 94.9 ft.

The angle of elevation of top = θ[tex]_1[/tex] = [tex]35.3^o[/tex]

The angle of elevation of bottom = θ[tex]_2=26.2^o[/tex]

If the height of building is [tex]x[/tex]

Then,

[tex]tan 26.2=x/y\\y=x/(tan26.2)\\[/tex]

And:

[tex]tan 35.3=(94.9+x)/y\\[/tex]

[tex]94.9+x=y \times tan35.3[/tex]

On putting the value in above equation:

[tex]x=\frac{94.9}{\frac{tan35.3}{tan26.2}-1 }[/tex]

solving the above equation:

[tex]x=214.84 ft.[/tex]

Hence, the height of the building is 214.84 ft.

For more information visit:

https://brainly.com/question/15274867?referrer=searchResults


Related Questions

This extreme value problem has a solution with both a maximum value and a minimum value. Use Lagrange multipliers to find the extreme values of the function subject to the given constraint. f(x, y) = x2 − y2; x2 + y2 = 81 maximum value minimum value

Answers

The Lagrangian is

[tex]L(x,y,\lambda)=x^2-y^2+\lambda(x^2+y^2-81)[/tex]

with critical points where the partial derivatives are identically zero:

[tex]L_x=2x+2\lambda x=0\implies 2x(1+\lambda)=0\implies x=0\text{ or }\lambda=-1[/tex]

[tex]L_y=-2y+2\lambda y=0\implies-2y(1-\lambda)=0\implies y=0\text{ or }\lambda=1[/tex]

[tex]L_\lambda=x^2+y^2-81=0\implies x^2+y^2=81[/tex]

If [tex]x=0[/tex], then [tex]L_\lambda=0\implies y=\pm9[/tex]If [tex]\lambda=-1[/tex], then [tex]L_y=0\implies y=0[/tex] and [tex]L_\lambda=0\implies x=\pm9[/tex]If [tex]y=0[/tex], we get the previous conclusion of [tex]x=\pm9[/tex]If [tex]\lambda=1[/tex], then [tex]L_x=0\implies x=0[/tex] and we again get [tex]y=\pm9[/tex]

So we have four critical points to consider: (0, -9), (0, 9), (-9,0), and (9, 0). We have

[tex]f(0,-9)=-81[/tex]

[tex]f(0,9)=-81[/tex]

[tex]f(-9,0)=81[/tex]

[tex]f(9,0)=81[/tex]

So the maximum value is 81 and the minimum value is -81.

The extreme values of a function are the minimum and the maximum values of the function.

The extreme values are -81 and 81

The function is given as:

[tex]\mathbf{f(x,y) = x^2 - y^2}[/tex]

[tex]\mathbf{x^2 + y^2 = 81}[/tex]

Subtract 81 from both sides of [tex]\mathbf{x^2 + y^2 = 81}[/tex]

[tex]\mathbf{x^2 + y^2 - 81 = 0}[/tex]

Using Lagrange multiplies, we have:

[tex]\mathbf{L(x,y,\lambda) = f(x,y) + \lambda(0)}[/tex]

Substitute [tex]\mathbf{f(x,y) = x^2 - y^2}[/tex] and [tex]\mathbf{x^2 + y^2 - 81 = 0}[/tex]

[tex]\mathbf{L(x,y,\lambda) = x^2 - y^2 + \lambda(x^2 + y^2 - 81)}[/tex]

Differentiate

[tex]\mathbf{L_x = 2x + 2\lambda x}[/tex]

[tex]\mathbf{L_y = -2y + 2\lambda y}[/tex]

[tex]\mathbf{L_{\lambda} = x^2 + y^2 -81}[/tex]

Equate to 0

[tex]\mathbf{2x + 2\lambda x = 0}[/tex]

[tex]\mathbf{-2y + 2\lambda y = 0}[/tex]

[tex]\mathbf{x^2 + y^2 -81 = 0}[/tex]

So, we have:

[tex]\mathbf{2\lambda x = -2x}[/tex]

[tex]\mathbf{2\lambda y = 2y}[/tex]

Divide both sides of [tex]\mathbf{2\lambda x = -2x}[/tex] by -2x

[tex]\mathbf{\lambda = -1}[/tex]

Divide both sides of [tex]\mathbf{2\lambda y = 2y}[/tex] by 2y

[tex]\mathbf{\lambda = 1}[/tex]

The above means that:

[tex]\mathbf{\lambda = -1\ or\ x = 0}[/tex]

[tex]\mathbf{\lambda = 1\ or\ y = 0}[/tex]

Recall that: [tex]\mathbf{x^2 + y^2 = 81}[/tex]

When x = 0, we have:

[tex]\mathbf{0^2 + y^2 = 81}[/tex]

Take square roots of both sides

[tex]\mathbf{y = \±9}[/tex]

When y = 0, we have:

[tex]\mathbf{x^2 + 0^2 = 81}[/tex]

Take square roots of both sides

[tex]\mathbf{x = \±9}[/tex]

To determine the critical points, we consider:

[tex]\mathbf{\lambda = -1\ or\ x = 0}[/tex] or [tex]\mathbf{y = \±9}[/tex]

[tex]\mathbf{\lambda = 1\ or\ y = 0}[/tex] or [tex]\mathbf{x = \±9}[/tex]

So, the critical points are:

[tex]\mathbf{(x,y) = \{ (0, -9), (0, 9), (-9,0), (9, 0)\}}[/tex]

Substitute the above values in [tex]\mathbf{f(x,y) = x^2 - y^2}[/tex]

[tex]\mathbf{f(0,-9) = 0^2 - (-9)^2 = -81}[/tex]

[tex]\mathbf{f(0,-9) = 0^2 - (9)^2 = -81}[/tex]

[tex]\mathbf{f(-9,0) = (-9)^2 - 0^2 = 81}[/tex]

[tex]\mathbf{f(9,0) = (9)^2 - 0^2 = 81}[/tex]

Considering the above values, we have:

[tex]\mathbf{Minimum= -81}[/tex]

[tex]\mathbf{Maximum= 81}[/tex]

Hence, the extreme values are -81 and 81, respectively.

Read more about extreme values at;

brainly.com/question/1286349

Customer account "numbers" for a certain company consist of 3 letters followed by 2 numbers.Step 1 of 2 : How many different account numbers are possible if repetitions of letters and digits are allowed?

Answers

Final answer:

If 3 letters are followed by 2 numbers and repetitions are allowed, there are a total of 1757600 possible different account numbers. Each letter position has 26 choices and each number position has 10 choices.

Explanation:

The student has asked to determine the number of different customer account "numbers" a company can have if the accounts consist of 3 letters followed by 2 numbers, with repetitions allowed for both letters and numbers.

To calculate the total number of possible account numbers, we can use the multiplication principle of counting. The number of options for each position of the account number is multiplied together to get the total number of combinations.

For the 3 letters, each position can contain any letter from A-Z, which gives us 26 choices per position. Since repeats are allowed, each of the 3 positions has 26 possible choices.

For the 2 numbers, each position can contain any digit from 0-9, which gives us 10 choices per position.

Therefore, to find the total number of possible account numbers, we calculate:

26 × 26 × 26 × 10 × 10 = 1757600 possible account numbers.

Riding a bike a person takes 20 minutes to go to work. The trip back home takes 30 minutes. If the rate back is 8 mph slower than the trip to work, find the rates (speeds) each way and the distance to work.

Answers

Answer:Trip to work has rate:  24 mphTrip back to home has rate: 18 mphDistance to work is:  480 mStep-by-step explanation:

We know that speed is defined as the ratio of distance to time.

i.e.

[tex]Speed=\dfrac{Distance}{Time}[/tex]

Let the distance traveled to work be: x m.

Now, while going to work it takes a person 20 minutes.

This means that the speed of the person while going to work is:

[tex]S_1=\dfrac{x}{20}[/tex]

Also, the time taken to come back home is: 30 minutes.

This means that the speed of person while riding to home is:

[tex]S_2=\dfrac{x}{30}[/tex]

Also, it is given that  the rate back is 8 mph slower than the trip to work.

This means that:

[tex]S_1-S_2=8[/tex]

i.e.

[tex]\dfrac{x}{20}-\dfrac{x}{30}=8\\\\i.e.\\\\\dfrac{30x-20x}{600}=8\\\\i.e.\\\\\dfrac{10x}{600}=8\\\\i.e.\\\\\dfrac{x}{60}=8\\\\i.e.\\\\x=480\ \text{m}[/tex]

Hence, the distance to work is:  480 m.

Also, the rate while going to work is:

[tex]=\dfrac{480}{20}\\\\=24\ \text{mph}[/tex]

and the trip back to home is covered with the speed:

[tex]=\dfrac{480}{30}\\\\=16\ \text{mph}[/tex]

Eliminate the parameter.

x = 3 cos t, y = 3 sin t

Answers

Answer:

x^2+y^2 = 3^2

Step-by-step explanation:

We need to eliminate the parameter t

Given:

x = 3 cos t

y = 3 sin t

Squaring the above both equations

(x)^2=(3 cos t)^2

(y)^2 =(3 sin t)^2

x^2 = 3^2 cos^2t

y^2=3^2 sin^2t

Now adding both equations

x^2+y^2=3^2 cos^2t+3^2 sin^2t

Taking 3^2 common

x^2+y^2=3^2 (cos^2t+sin^2t)

We know that cos^2t+sin^2t = 1

so, putting the value

x^2+y^2=3^2(1)

x^2+y^2 = 3^2

Hence the parameter t is eliminated.

Final answer:

To eliminate the parameter in the given equations x = 3 cos t and y = 3 sin t, we can substitute cos(t) and sin(t) in terms of x and y to eliminate the parameter. The resulting equations represent the line y = x.

Explanation:

To eliminate the parameter in the given equations x = 3 cos t and y = 3 sin t, we need to express x and y in terms of each other without the parameter 't'. Using the identity cos^2(t) + sin^2(t) = 1, we can solve for cos(t) and sin(t), and substitute them into the equations to eliminate the parameter.

Using the fact that cos(t) = x/3 and sin(t) = y/3, we can rewrite the equations as x = 3 cos(t) = 3(x/3) = x and y = 3 sin(t) = 3(y/3) = y. Therefore, eliminating the parameter results in x = x and y = y, which simply means that the equations represent the line y = x.

Learn more about Eliminating parameters from equations here:

https://brainly.com/question/32043775

#SPJ3

The laws shown in the video—kirchhoff’s junction law and kirchhoff’s loop law—are not newly introduced laws of physics. The junction law is based on __________

Answers

Answer:

  The junction law is based on the conservation of charge.

Step-by-step explanation:

Kirchhoff's current law, or junction law, (1st Law) states that current flowing into a node (or a junction) must be equal to current flowing out of it. This is a consequence of charge conservation—charge is not created or destroyed in a closed system.

Which of the following is not listed as natural causes for our annihilation? OA. Super Volcano B. Asteroid Impact C. Gamma Rays D. Epidemic E. None of the above

Answers

Answer:

its E none of the above

Step-by-step explanation:

A customer brings a check of 2,941. he wants 100 in cash, put 20% of the remaining into her savings account then the rest into a checking account. How much will ge be putting in his checking account

Answers

Answer:

Amount theta she is putting in Checking account is 2272.80

Step-by-step explanation:

Given:

Amount on check = 2941

Amount that he want in cash = 100

Amount she put in saving account = 20% of remaining after getting cash

Remaining Amount she put in checking account.

To find: Amount in her Checking Account.

Amount left after taking cash = 2941 - 100 = 2841

Amount that she put in saving account = 20% of 2841 = [tex]\frac{20}{100}\times2841[/tex] =  568.20

Amount in her checking account = 2941 - 100 - 568.20  = 2272.8

Therefore, Amount theta she is putting in Checking account is 2272.80

Inclusions are defects in poured metal caused by contaminants. The number of (large) inclusions in cast iron follows a Poisson distribution with a rate of 3.2 per cubic millimetre. What is the probability of exactly four inclusions in 2.0 cubic millimetres? Please enter the answer to 3 decimal places.

Answers

Answer: 0.116

Step-by-step explanation:

The Poisson distribution probability formula is given by :-

[tex]P(X=x)=\dfrac{e^{-\lambda}\lambda^x}{x!}[/tex], where \lambda is the mean of the distribution and x is the number of success

Given : The number of inclusions in one cubic millimeter = 3.2

Then , the number of inclusions in two cubic millimeters=[tex]\lambda=2\times3.2=6.4[/tex]

Now, the probability of exactly four inclusions in 2.0 cubic millimetres is given by :-

[tex]P(X=4)=\dfrac{e^{-6.4}(6.4)^4}{4!}\\\\=0.11615127195\approx0.116[/tex]

Hence, the probability of exactly four inclusions in 2.0 cubic millimetres = 0.116

3. For each of the following lists of integers, provide a simple formula or rule.. Assuming that your formula or rule is correct, determine the next three term of the sequence. 15, 20, 25, 30, 35,... a. b. 5,9, 13, 17, 21, ...

Answers

Step-by-step explanation:

Consider the first sequence:

15, 20, 25, 30, 35,...

Note that each term is increased by 5 from its previous term.

Therefore,

[tex]a_n=a_{n-1}+5[/tex]

If the pattern continue, the next three term of the sequence will be:

[tex]a_6=a_{6-1}+5[/tex]

[tex]a_6=a_{5}+5[/tex]

[tex]a_6=35+5[/tex]

[tex]a_6=40[/tex]

Similarly,

[tex]a_7=a_{7-1}+5[/tex]

[tex]a_7=a_{6}+5[/tex]

[tex]a_7=40+5[/tex]

[tex]a_7=45[/tex]

Similarly,

[tex]a_8=a_{8-1}+5[/tex]

[tex]a_8=a_{7}+5[/tex]

[tex]a_8=45+5[/tex]

[tex]a_8=50[/tex]

Thus, the next three term of the sequence 15, 20, 25, 30, 35,... is 40, 45, and 50.

Now, consider the second sequence:

5, 9, 13, 17, 21,...

Note that each term is increased by 4 from its previous term.

Therefore,

[tex]a_n=a_{n-1}+4[/tex]

If the pattern continue, the next three term of the sequence will be:

[tex]a_6=a_{6-1}+4[/tex]

[tex]a_6=a_{5}+4[/tex]

[tex]a_6=21+4[/tex]

[tex]a_6=25[/tex]

Similarly,

[tex]a_7=a_{7-1}+4[/tex]

[tex]a_7=a_{6}+4[/tex]

[tex]a_7=25+4[/tex]

[tex]a_7=29[/tex]

Similarly,

[tex]a_8=a_{8-1}+4[/tex]

[tex]a_8=a_{7}+4[/tex]

[tex]a_8=29+4[/tex]

[tex]a_8=33[/tex]

Thus, the next three term of the sequence 5, 9, 13, 17, 21,... is 25, 29, and 33.


How many primes are between 2^31 and 2^32? What is the approximate ratio of primes to
all numbers between 2^31 and 2^32?

Answers

Final answer:

There are approximately 132489 prime numbers between 2³¹and 2³², with a ratio of primes to all numbers being approximately 0.1156.

Explanation:

To find the number of primes between 2³¹and 2³², we can use the Sieve of Eratosthenes algorithm. With this algorithm, we can mark all the multiples of each prime number, and the remaining unmarked numbers will be prime.

Using this method, we can calculate that there are approximately 132489 primes between 2³¹ and 2³². The ratio of primes to all the numbers between 2³¹and 2³²is approximately 0.1156.

Trucks in a delivery fleet travel a mean of 120 miles per day with a standard deviation of 22 miles per day. The mileage per day is distributed normally. Find the probability that a truck drives between 100 and 157 miles in a day. Round your answer to four decimal places.

Answers

Final answer:

The probability that a truck drives between 100 and 157 miles in a day within a normal distribution can be calculated using z-scores. The z-scores for 100 and 157 miles are computed relative to the mean and standard deviation, and the corresponding probabilities are obtained from the standard normal distribution table. The final probability is the difference of these two probabilities.

Explanation:

Given that the distribution of trucks' daily mileage is normally distributed, we can approach this problem by using the principles of normal distribution and z-scores. The z-score is a measure of how many standard deviations an element is from the mean.

First, we calculate the z-scores for both 100 miles and 157 miles:

Z1 =(100 - 120) / 22 = -0.9091 Z2 = (157 - 120) / 22 = 1.6818

Next, we look up these z-scores in the standard normal distribution table (or use a calculator with a normal distribution function), which will give us the probabilities P(Z To arrive at four decimal places precision, this process typically involves using a statistical calculator or software.

Learn more about Normal Distribution here:

https://brainly.com/question/34741155

#SPJ2

Find two power series solutions of the given differential equation about the ordinary point x = 0. Compare the series solutions with the solutions of the differential equation obtained using the method of Section 4.3. Try to explain any differences between the two forms of the solution. y'' − y' = 0

Answers

Final answer:

To find power series solutions of the differential equation y'' − y' = 0, we can assume a power series solution and find the recurrence relation. Two power series solutions are found by choosing different initial conditions. The power series solutions are equivalent to the exponential solutions obtained using another method.

Explanation:

To find power series solutions of the differential equation y'' − y' = 0, we can assume a power series solution of the form y(x) = ∑(n=0)∞ a_nx^n. Substituting this into the differential equation and simplifying, we find that the power series satisfies the recurrence relation a_{n+2} = a_{n+1} in terms of a_0 and a_1.

By letting a_0 = 0 and a_1 = 1, we obtain the power series solution y_1(x) = x. Alternatively, by letting a_0 = 1 and a_1 = 0, we obtain the power series solution y_2(x) = 1.

Comparing these power series solutions with the solutions obtained using the method of Section 4.3, we see that the power series solutions are polynomials. In this case, the power series solutions are equivalent to the solutions obtained using the method of Section 4.3, which are exponential functions.

Learn more about Power series solution here:

https://brainly.com/question/31522450

#SPJ11

You would like to make a salad that consists of​ lettuce, tomato,​ cucumber, and onions. You go to the supermarket intending to purchase one variety of each of these ingredients. You discover that there are nine varieties of​ lettuce, four varieties of​ tomatoes, two varieties of​ cucumbers, and three varieties of onions for sale at the supermarket. How many different salads can you​ make?

Answers

Answer:  216

Step-by-step explanation:

Given : We like to make a salad that consists of​ lettuce, tomato,​ cucumber, and onions.

The number of varieties of lettuce = 9

The number of varieties of tomatoes = 4

The number of varieties of cucumbers = 2

The number of varieties of onions = 3

Now, the number of different salads we can make is given by :-

[tex]9\times4\times2\=216[/tex]

Hence, we can make 216 different types of salads.

Please help me with this

Answers

Answer:

Definition of mid-point

Step-by-step explanation:

Midpoint is the center point of a line segment

At midpoints the line segment is divided into two equal lengths

Point K is the midpoint of segment MJ and length of segment MK =  length of segment KJ

Point K is the midpoint of segment OL and length of segment OK = length of segment KL

An 80 kg Rottweiler needs 40 mL/kg over 12 hours. What's the flow rate per hour?


A. 3,200 mL/hr
B. 267 mL/hr
C. 3.3 mL/hr
D. 133 mL/hr

Answers

Answer:

The flow rate is 267ml/hour

Step-by-step explanation:

Hello, great question. These types are questions are the beginning steps for learning more advanced Equations.

To solve this we first need to find out how many ml the Rottweiler needs over 12 hours. We do this by using the Rule of Three property.

[tex]\frac{40ml}{1kg} = \frac{x}{80kg}[/tex]

[tex]\frac{40ml*80kg}{1kg} =x[/tex]

[tex]3200ml = x[/tex]

So the Rottweiler needs 3200 ml over a 12 hour period. We now need to find the flow rate per hour. We can solve this by simply dividing 3200 ml by 12 hours.

[tex]3200ml / 12hours = 266.67ml/hour[/tex]

So the flow rate is 267 ml/hour (rounded to the nearest whole number)

I hope this answered your question. If you have any more questions feel free to ask away at Brainly.

An english reading list has 9 american novels and 7 english novels. A student must read 5 from the list and at least 3 must be english novels. In how many different ways can the five books be selected combination

Answers

[tex]_7C_3\cdot {_{13}C_2}=\dfrac{7!}{3!4!}\cdot\dfrac{13!}{2!11!}=\dfrac{5\cdot6\cdot7}{2\cdot3}\cdot\dfrac{12\cdot13}{2}=2730[/tex]

Final answer:

The number of ways the student can select 5 books such that at least 3 are English novels can be calculated as the sum of combinations of 3 English and 2 American, 4 English and 1 American, and all 5 being English.

Explanation:

The subject matter of this question is based in the mathematics field, specifically combinatorics. To tackle this problem, we will utilize the concept of combination, which is a way of selecting items from a larger set where order does not matter.

The student has to select 5 books out of 16 (9 American and 7 English novels). But at least 3 should be English novels. It means the student can pick 3, 4 or all 5 novels as English novels. Let's calculate each possibility:

3 English novels and 2 American novels: Ways = C(7,3) * C(9,2) 4 English novels and 1 American novel: Ways = C(7,4) * C(9,1) All 5 English novels: Ways = C(7,5)

So, the total number of ways = [C(7,3)*C(9,2)] + [C(7,4)*C(9,1)] + C(7,5). Here C(n,r) denotes combination and is equal to n! / [(n-r)!*r!], where '!' denotes factorial.

Learn more about Combinations here:

https://brainly.com/question/37999460

#SPJ2

Which is the solution to the equation 3/5(x+4/3)= 1.04 ?

Answers

The correct answer is 2/5

3/5X + 4/5 =26/25

15x+20=26

15x=26-20

15x=6

x=2/5

Find the value of 715×211 Although these numbers aren't quite as nice as the ones from the example, the procedure is the same, so the difficulty is the same same excepting the ability to perform the calculation in your head. You may choose to use a calculator.

Answers

Final answer:

To calculate the value of 715 × 211, you can use the standard multiplication method by multiplying each digit of the two numbers and summing up the results.

Explanation:

To find the value of 715 × 211, you can use the standard multiplication method. Start by multiplying the ones digit of 715 (5) by each digit of 211 (1, 1, and 2), and write down the results. Then, multiply the tens digit of 715 (1) by each digit of 211, and write down the results one place to the left of the previous results. Finally, multiply the hundreds digit of 715 (7) by each digit of 211 and write down the results two places to the left. Sum up the columns and you will get the final product.

Here's how it looks:

   715
× 211
--------
 715
 1430  
+1425
--------
 150665

Learn more about Multiplication here:

https://brainly.com/question/35502092

#SPJ2

A medical equipment industry manufactures X-ray machines. The unit cost C (the cost in dollars to make each X-ray machine) depends on the number of machines made. If X machines are made, then the unit cost is given by the function C (x) = 1.2x^2 -600x + 89,966. How many machines must be made to minimize the unit cost?
Do not round your answer.

Answers

Answer:

x = 250 units

Step-by-step explanation:

We can easily solve this problem by using a graphing calculator or any plotting tool.

We must find the minimum point in the graph. This corresponds to the number of machines that produce the minimum cost.

The equation is

C (x) = 1.2x^2 -600x + 89,966

Please see attached image below

By producing x = 250 units, we obtain the minimum cost

Write the linear system of differential equations in matrix form then solve the system.

dx/dt = x + y

dy/dt = 4x + y

x(0) = 1, y(0) = 2

Answers

In matrix form, the system is

[tex]\dfrac{\mathrm d}{\mathrm dt}\begin{bmatrix}x\\y\end{bmatrix}=\begin{bmatrix}1&1\\4&1\end{bmatrix}\begin{bmatrix}x\\y\end{bmatrix}[/tex]

First find the eigenvalues of the coefficient matrix (call it [tex]\mathbf A[/tex]).

[tex]\det(\mathbf A-\lambda\mathbf I)=\begin{vmatrix}1-\lambda&1\\4&1-\lambda\end{vmatrix}=(1-\lambda)^2-4=0\implies\lambda^2-2\lambda-3=0[/tex]

[tex]\implies\lambda_1=-1,\lambda_=3[/tex]

Find the corresponding eigenvector for each eigenvalue:

[tex]\lambda_1=-1\implies(\mathbf A+\mathbf I)\vec\eta_1=\vec0\implies\begin{bmatrix}2&1\\4&2\end{bmatrix}\begin{bmatrix}\eta_{1,1}\\\eta_{1,2}\end{bmatrix}=\begin{bmatrix}0\\0\end{bmatrix}[/tex]

[tex]\lambda_2=3\implies(\mathbf A-3\mathbf I)\vec\eta_2=\vec0\implies\begin{bmatrix}-2&1\\4&-2\end{bmatrix}\begin{bmatrix}\eta_{2,1}\\\eta_{2,2}\end{bmatrix}=\begin{bmatrix}0\\0\end{bmatrix}[/tex]

[tex]\implies\vec\eta_1=\begin{bmatrix}1\\-2\end{bmatrix},\vec\eta_2=\begin{bmatrix}1\\2\end{bmatrix}[/tex]

Then the system has general solution

[tex]\begin{bmatrix}x\\y\end{bmatrix}=C_1\vec\eta_1e^{\lambda_1t}+C_2\vec\eta_2e^{\lambda_2t}[/tex]

or

[tex]\begin{cases}x(t)=C_1e^{-t}+C_2e^{3t}\\y(t)=-2C_1e^{-t}+2C_2e^{3t}\end{cases}[/tex]

Given that [tex]x(0)=1[/tex] and [tex]y(0)=2[/tex], we have

[tex]\begin{cases}1=C_1+C_2\\2=-2C_1+2C_2\end{cases}\implies C_1=0,C_2=2[/tex]

so that the system has particular solution

[tex]\begin{bmatrix}x\\y\end{bmatrix}=\begin{bmatrix}e^{3t}\\2e^{3t}\end{bmatrix}[/tex]

Final answer:

The linear system of differential equations can be written in matrix form as [dx/dt, dy/dt] = [1, 1; 4, 1] * [x, y]. By solving the system with the given initial conditions x(0) = 1 and y(0) = 2, the values of x and y at different time points can be determined.

Explanation:

To write the linear system of differential equations in matrix form, we can express the given equations as:

[dx/dt, dy/dt] = [1, 1; 4, 1] * [x, y]

Using the initial conditions x(0) = 1 and y(0) = 2, we can solve the system of equations to find the values of x and y at different time points.

Automobile sales in a country were 20.6 million thisâ year, a 4.9â% increase over last year. Find the number of auto sales in the country last year.?

number â(Round to the nearestâ tenth.)?

Answers

Answer: 19.6 million

Step-by-step explanation:

The exponential growth function is given by :-

[tex]A=A_0(1+r)^x[/tex], where A is the initial amount , r is rate of interest and x is time period.

Given : The automobile sales in a country this year : A= 20.6 million

The rate of increase : r = 4.9 %=0.049

For last year , we take x = 1 , then the required exponential equation will be :-

[tex]20.6=A_0(1+0.049)^1\\\\\Rightarrow\ A_0=\dfrac{20.6}{1.049}=19.63775\approx19.6[/tex]

Hence, the  number of auto sales in the country last year = 19.6 million.

Final answer:

To find last year's auto sales, the formula original amount = final amount / (1 + rate of increase) is used. The sales last year, before a 4.9% increase to 20.6 million, were approximately 19.6 million when rounded to the nearest tenth.

Explanation:

To find the number of automobile sales last year before the increase, we can use the formula: original amount = final amount / (1 + rate of increase).

Given that the sales this year were 20.6 million and the rate of increase was 4.9%, the calculation for last year's sales would be as follows:

Original sales = 20.6 million / (1 + 0.049) = 20.6 million / 1.049

After performing the division, we get:

Original sales = 19.638 million

Rounding to the nearest tenth, the number of auto sales in the country last year was 19.6 million.

A normal distribution has a mean 20 and standard deviation 5. What is the z score that corresponds to the value x=222

Answers

Answer: The z score that corresponds to the value x=22 is 0.4 .

Step-by-step explanation:

Given : A normal distribution has a mean 20 and standard deviation 5.

i.e. [tex]\mu=20[/tex]

[tex]\sigma=5[/tex]

Let x be the random selected variable.

We know that to find the z-score corresponds to the value x is given by :-

[tex]z=\dfrac{x-\mu}{\sigma}[/tex]

For x = 22, we have

[tex]z=\dfrac{22-20}{5}=\dfrac{2}{5}\\\\\Rightarrow\ z=0.4[/tex]

Hence, the z score that corresponds to the value x=22 is 0.4

A z-score in a normal distribution measures the number of standard deviations a value is from the mean. To calculate it, use the formula z = (x - μ) / σ for the specific values provided, such as half a standard deviation below the mean, 5 points above the mean, three standard deviations above the mean, and 22 points below the mean.

The calculation of a z-score within a normal distribution is a common task in statistics, allowing one to determine how many standard deviations a particular value, x, is from the mean, μ, of the distribution. The z-score is calculated using the formula:

z = (x - μ) / σ

where x is the value in question, μ is the mean, and σ is the standard deviation. Now, we will calculate the z-scores for the given situations:

One-half of a standard deviation below the mean:5 points above the mean:Three standard deviations above the mean:22 points below the mean:

Remember, when you use these calculations for specific numerical values, you need to insert the actual values of mean and standard deviation into the formula.

How do you simplify this?

[tex](9k^{6}+8k^{4}-6k^{2})(4k^{2}-5)[/tex]

Answers

ANSWER

[tex]36k^{8} -13{k}^{6} -64k^{4} + 30 {k}^{2} [/tex]

EXPLANATION

Recall the distributive property:

[tex](a + b + c)(d + e) = a(d + e) + b(d + e) + c(d + e)[/tex]

We apply this property multiple times to simplify

[tex](9k^{6}+8k^{4}-6k^{2})(4k^{2}-5)[/tex]

This implies that:

[tex]9k^{6}(4k^{2}-5)+8k^{4}(4k^{2}-5)-6k^{2}(4k^{2}-5)[/tex]

We apply the distributive property again:

This time: a(b+c)=ac+ab

[tex] \implies \: 9k^{6} \times 4k^{2}-5 \times 9 {k}^{6} +8k^{4} \times 4k^{2}-5 \times 8 {k}^{4} -6k^{2} \times 4k^{2} + 5 \times 6 {k}^{2} [/tex]

[tex]\implies \: 36k^{8} -45{k}^{6} +32k^{6} -40 {k}^{4} -24k^{4} + 30 {k}^{2} [/tex]

[tex]\implies 36k^{8} -13{k}^{6} -64k^{4} + 30 {k}^{2} [/tex]

NB: [tex]k^{n}\times{k}^{m}=k^{m+n} [/tex]

2) Here are two relations defined on the set {a, b, c, d): S= { (a, b), (a, c), (c, d), (c, a)} R={ (b, c), (c, b), (a, d), (d, b)} Write each relation as a set of ordered pairs. a) SoR b) RoS c) SoS

Answers

Answer:

Given relations defined on the set {a, b, c, d},

S= { (a, b), (a, c), (c, d), (c, a)}

R={ (b, c), (c, b), (a, d), (d, b)},

Since, SoR(x) = S(R(x)),

So, SoR(a) = S(R(a)) = S(d) = ∅,

SoR(b) = S(R(b)) = S(c) = d and a,

SoR(c) = S(R(c)) = S(b) = ∅,

SoR(d) = S(R(d)) = S(b) = ∅,

Thus, SoR = { (b,d), (b,a) }

RoS(a) = R(S(a)) = R(b) = c and RoS(a) = R(S(a)) = R(c) = b,

RoS(b) = R(S(b)) = R(∅) = ∅,

RoS(c) = R(S(c)) = R(d) = b and RoS(c) = R(S(c)) = R(a) = d

RoS(d) = R(S(d)) = R(∅) = ∅,

Thus, RoS = { (a, c), (a, b), (c,d), (c, b) },

SoS(a) = S(S(a)) = S(b) = ∅ and SoS(a) = S(S(a)) = S(c) = d and a

SoS(b) = S(S(b)) = S(∅) = ∅,

SoS(c) = S(S(c)) = S(d) = ∅ and SoS(c) = S(S(c)) = S(a) = b and c

SoS(d) = S(S(d)) = S(∅) = ∅,

SoS = { (a, d), (a, a), (c, b), (c, c) }

Final answer:

The composition of relations S and R mentioned in the question are SoR: { (a, c), (c, b)}, RoS: { (b, d), (a, b)} and SoS: { (a, d), (c, b)}.

Explanation:

The question is asking for the composition of relations. So, composition of relations S and R, denoted as 'SoR' or 'S ◦ R', is the set of ordered pairs where the first element is related to the second element through the combination of relations S and R. In this case the relations S and R on the set {a, b, c, d} are: S= { (a, b), (a, c), (c, d), (c, a)} and R={ (b, c), (c, b), (a, d), (d, b)}.

By the rule of composition SoR will be: { (a, c), (c, b)}.

Similarly, for RoS will be: { (b, d), (a, b)}.

And for SoS it will be: { (a, d), (c, b)}.

Learn more about Composition of Relations here:

https://brainly.com/question/11269249

#SPJ3

The tread life of tires mounted on light duty trucks follows the normal probability distribution with a mean of 60,000 miles and a standard deviation of 4,000 miles. Suppose you bought a set of four tires, what is the likelihood the mean tire life of these four tires is more than 66,000 miles?

Answers

Answer:  0.0013

Step-by-step explanation:

Given : The test scores are normally distributed with

Mean : [tex]\mu=\ 60,000[/tex]

Standard deviation :[tex]\sigma= 4,000[/tex]

Sample size : [tex]n=4[/tex]

The formula to calculate the z-score :-

[tex]z=\dfrac{x-\mu}{\dfrac{\sigma}{\sqrt{n}}}[/tex]

For x = 66,000

[tex]z=\dfrac{66000-60000}{\dfrac{4000}{\sqrt{4}}}=3[/tex]

The p-value = [tex]P(z>3)\=1-P(z<3)=1- 0.9986501\approx0.0013[/tex]

Hence, the likelihood the mean tire life of these four tires is more than 66,000 miles = 0.0013

Find the remainder when dividing 2^2013 by 15.

*Answer should be in modulo. Example: Find the remainder when dividing 2^100 by 21 and the answer is 2^100 = 16mod(21).*

Answers

[tex]2^{2013}=2^{4\cdot503+1}\\\\2^4=16\equiv 1\pmod{15}\\2^{4\cdot 503}\equiv 1\pmod{15}\\2^{4\cdot 503+1}\equiv 2\pmod{15}\\\\2^{2013}\equiv 2\pmod{15}[/tex]

Prove that (AB)−1=B−1A−1

Answers

Answer with Step-by-step explanation:

Consider,

[tex](AB)^{-1}(AB)=I[/tex] (Identity rule)

Multiplying by B⁻¹ on the both the sides, we get that

[tex](AB)^{-1}(AB)B^{-1}=IB^{-1}\\\\(AB)^{-1}A(BB^{-1})=B^{-1}[/tex]

And we know that BB⁻¹ = I

So, it becomes,

[tex](AB)^{-1}A=B^{-1}[/tex]

Now, multiplying by A⁻¹ on both the sides, we get that

[tex](AB)^{-1}AA^{-1}=B^{-1}A^{-1}\\\\(AB)^{-1}=B^{-1}A^{-1}[/tex] (AA⁻¹=I)

Hence, proved.


A vacuum cleaner dealership sold 370 units in 2011 and 411 units in 2012. Find the percent increase or decrease in the number of units sold.

The number of units sold increased or decreased? by about what percent?

Answers

Answer:

The percent of Increase is of 11.08% (0.1108)

Step-by-step explanation:

Hello, great question. These types are questions are the beginning steps for learning more advanced Algebraic Equations.

Since we have two different values for two different years, we can use the following Algebraic Expression to calculate the percent difference of sales between both years. The Expression would be the following,

[tex]370 * (x+1) = 411[/tex]

Where x is the percent difference. Now we solve for x,

[tex]370 * (x+1) = 411[/tex]

[tex]x+1 = 1.1108[/tex]

[tex]x = 0.1108[/tex]

so now we see that the percent of Increase is of 11.08% (0.1108)

I hope this answered your question. If you have any more questions feel free to ask away at Brainly.

Final answer:

The number of units sold increased by about 11.08%.

Explanation:

To find the percent increase or decrease in the number of units sold, we need to calculate the difference between the number of units sold in 2012 and 2011, and then divide that difference by the number of units sold in 2011.

The amount of increase or decrease is calculated as: (Number of units sold in 2012 - Number of units sold in 2011)/Number of units sold in 2011 x 100

In this case, the calculation is: (411 - 370)/370 x 100 = (41/370) x 100 = 11.08%

Therefore, the number of units sold increased by about 11.08%.

A loan of $1000 is to be paid back, with interest, at the end of 1 year. Aft er 3 months, a partial payment of $300 is made. Use the US Rule to determine the balance due at the end of one year, considering the partial payment. Assume a simple interest rate of 9%.

Answers

Answer:

total balance due at the end of 1 year is  $769.75

Step-by-step explanation:

Given data

loan amount = $1000

time period = 1 year

return = $300

rate = 9%

to find out

balance due at the end of one year

solution

we know in question $300 return after 3 month so we first calculate interest of $1000 for 3 month and than we after 3 month remaining 9 month we calculate interest for $700

interest for first 3 month = ( principal × rate × time ) / 100    .............1

here time is 3 month so = 3/12  will take and rate 9 % and principal $1000

put all these value in equation 1 we get interest for first 3 month

interest for first 3 month = ( principal × rate × time ) / 100

interest for first 3 month = ( 1000 × 9 × 3/12 ) / 100

interest for first 3 month = $22.5

now we calculate interest for remaining 9 months i.e.

interest for next 9 months = ( principal × rate × time ) / 100

here principal will be $700 because we pay $300 already

interest for next 9 months = ( 700 × 9 × 9/12 ) / 100

interest for next 9 months = $47.25

now we combine both interest that will be

interest for first 3 months +interest for next 9 months = interest of 1 year

interest of 1 year  =   $22.5  + $47.25

interest of 1 year  =   $69.75

so amount will be paid after 1 year will be loan amount + interest

amount will be paid after 1 year = 1000 + 69.75

amount will be paid after 1 year is $1069.75

so total balance due at the end of 1 year = amount will be paid after 1 year - amount paid already

total balance due at the end of 1 year = $1069.75 - $300

total balance due at the end of 1 year is  $769.75

Write out the form of the partial fraction decomposition of the function (as in this example). Do not determine the numerical values of the coefficients. (a) x4 − 2x3 + x2 + 3x − 2 x2 − 2x + 1

Answers

[tex]\dfrac{x^4-2x^3+x^2+3x-2}{x^2-2x+1}[/tex]

The degree of the numerator exceeds the degree of the denominator, so first you have to divide:

[tex]x^2+\dfrac{3x-2}{x^2-2x+1}[/tex]

Now, [tex]x^2-2x+1=(x-1)^2[/tex], so the remainder term can be expanded to get

[tex]\boxed{x^2+\dfrac a{x-1}+\dfrac b{(x-1)^2}}[/tex]

Other Questions
Identify the zeros of f(x) = (x + 1)(x 8)(5x + 2). 1, 2 over 5, 8 1, 2 over 5,8 1, 2 over 5, 8 1, 2 over 5,8 An array of int named a that contains exactly five elements has already been declared and initialized. In addition, an int variable j has also been declared and initialized to a value somewhere between 0 and 3. assign the programming symbols please help thanks attachment linked Which of these is a risk of speeding? A tire damageB greater damage on impact C greater likelihood of Being distracted D mechanical failure Find the equation of the line passing through (4, 3) and (-2, -7): A. 3x + 5y = 27 B. 3x - 5y = -3 C. 5x - 3y = 11 D. 2x - y = 5 A 14-year-old boy has been brought to the emergency department by his mother in excruciating pain that is radiating from his scrotum to his inguinal area. The boys heart rate is 122 beats per minute and he has vomited twice before arrival at the hospital. Examination reveals that his scrotum is reddened and slightly swollen and the testes are firm to touch and tender, with extensive cremaster muscle contraction noted. What is the boys most likely diagnosis? What is the truth value for the following conditional statement? p: true q: false q p T F F T T F F T T T F T Read the following passage from Bulfinchs Mythology:The Greek poets believed the earth to be flat and circular, their own country occupying the middle of it, the central point being either Mount Olympus, the abode of the gods, or Delphi, so famous for its oracle.Based on this passage, a reader could infer that the ancient Greek poets:were conceitedwere inventivetravelled locallytravelled abroad A 12-ft circular steel rod with a diameter of 1.5-in is in tension due to a pulling force of 70-lb. Calculate the stress in the rod A manufacturing machine has two processes. One of them is repeated 4 times and the second only once. The entire cycle can take no longer than 3 minutes. Which graph represents this scenario? Identify the fluid filled space between the cornea and iris. 7+3 to the second power+(12-8) divided by 2x 4 is The defendant was on trial for murder. The defendant called a witness to testify to an alibi. On cross-examination of the witness, the prosecutor asked, "Weren't you on the jury that acquitted the defendant of another criminal charge?"What is the best reason for sustaining an objection to this question?A The question goes beyond the scope of direct examination.B The probative value of the answer would be substantially outweighed by its tendency to mislead.C The question is a leading question.D Prior jury service in a case involving a party renders the witness incompetent. Which expression is equivalent to 30 (1/2 x - 2) + 40(3/4 y-4) CAN SOMEONE HELP ME WITH THIS MATH QUESTION ITS ABOUT TRANSLATION A cylindrical specimen of some metal alloy 11.2 mm (0.4409 in.) in diameter is stressed elastically in tension. A force of 14100 N (3170 lbf) produces a reduction in specimen diameter of 7 10-3 mm (2.756 10-4 in.). Compute Poisson's ratio for this material if its elastic modulus is 100 GPa (14.5 106 psi). What is a question you can ask when analyzing the structure of a text? How are allusions used for effect? How are main ideas developed? How are the examples effective? How are the points organized? A certain sound level increases by an additional 20 db. By how much does the intensity increase? A 0.43-kg object mass attached to a spring whose spring constant is 561 N/m executes simple harmonic motion. If its maximum speed is 8 m/s, what is the amplitude of its oscillation (in m)? Round your answer to the nearest tenth. A tire manufacturer produces 400 tires valued at $20 each. Three hundred tires are sold to a tire shop, which then sells them to households for $50 each. The remaining tires are unsold and are added to the tire manufacturers inventory. How much is added to GDP? a. $8,000 b. $15,000 c. $13,000 d. $17,000